Your-Doctor
Multiple Choice Questions (MCQ)


Quiz Categories Click to expand

Category: Q&A Medicine--->Infectious Diseases
Page: 3

Question 11# Print Question

A 63-year-old man presents to his physician due to persistent fevers. He says that he initially saw a physician for a fever about a month ago, and the workup was negative. He did not receive antibiotics at that time. Since then, he has measured his temperature at home and has had temperatures higher than 38.5°C. He denies any headache, cough, shortness of breath, abdominal pain, jaundice, bone or muscle pain, body swelling, or other concerning symptoms. He is otherwise healthy and sees the physician annually, receiving all necessary screening procedures. He has not traveled recently and has had no sick contacts or animal exposures. On physical examination, a new murmur is heard over the cardiac apex, but the rest of the examination, including assessment of skin and lymph nodes, is unremarkable. Three sets of blood cultures are sent to the laboratory and later return negative. An echocardiogram is performed and shows a vegetation on the mitral valve. Other significant laboratory values include a hemoglobin of 12.4 g/dL, a leukocyte count of 11,500/mm3 , and negative testing for ANA and antiphospholipid antibodies.

Which of the following is the likely cause of this patient’s recurring fevers?

A. Malignancy
B. Verrucous endocarditis
C. Infective endocarditis
D. Large artery vasculitis


Question 12# Print Question

A 54-year-old woman with a history of rheumatoid arthritis and hypertension presents to the Emergency Department with a high fever and headache. The symptoms began last night and were worse this morning. She also complains of severe sensitivity to light. Her medications include hydrochlorothiazide and methotrexate. She denies any cough, weight loss, chest pain, shortness of breath, or abdominal pain, and she has no history of chronic headaches or neurologic disease. Her temperature is 39.5°C, blood pressure is 146/90 mmHg, heart rate is 88 beats per minute, and respiratory rate is 14 breaths per minute. She is somewhat confused, and there is significant nuchal rigidity and her headache becomes worse when she moves her head quickly to the side. The neurologic examination, including a cranial nerve examination, is unremarkable. A funduscopic examination shows bilateral blurring of the optic disk margins and retinal venous engorgement.

Which of the following should be performed next in the workup of this patient?

A. CT scan
B. MRI
C. Lumbar puncture
D. Administer empiric antibiotics


Question 13# Print Question

A 33-year-old woman presents with a chief complaint of abdominal pain. The pain is epigastric and is worse with meals. It is associated with nausea, a burning sensation, and bloating. She denies any weight loss, vomiting, and bloody or dark stools. She has no other medical problems and takes no medications. She denies recent NSAID use or heavy alcohol consumption. An upper endoscopy is performed, which shows several gastric ulcers and one duodenal ulcer. Biopsy confirms infection with Helicobacter pylori.

What is the most appropriate treatment at this time?

A. Amoxicillin, metronidazole, and bismuth
B. Amoxicillin, clarithromycin, and omeprazole
C. Lansoprazole, bismuth, metronidazole, and tetracycline
D. Symptomatic treatment with antacids


Question 14# Print Question

A 43-year-old woman presents to her physician for severe joint pain affecting her hands, elbows, and left knee. She has no other medical problems and works as a medical assistant at a local prison. On examination, the metacarpophalangeal joints of both hands are swollen and erythematous. Blood work reveals a positive rheumatoid factor. She elects to begin treatment with a disease-modifying antirheumatic drug (DMARD). Several weeks later, the patient has an improvement of the pain in her joints but now complains of fever, chills, and a productive cough. Her chest x-ray is shown in Figure below.

Which of the following represents the mechanism of action of this patient’s medication?

A. Binds CD20 on the surface of B-cells
B. Binds and inhibits the effects of tumor necrosis factor-α (TNF-α)
C. Competitively inhibits dihydrofolate reductase
D. Nonselectively inhibits both cyclooxygenase-1 and -2 (COX-1, -2)


Question 15# Print Question

A 32-year-old mailman presents to the Emergency Department with the development of fever and shortness of breath over the past day. He denies any sick contacts or abnormal exposures and was previously well with no past medical history. He is tachycardic and tachypneic, and a chest x-ray of his lungs shows mediastinal widening and bilateral pleural effusions.

Which of the following is the most likely responsible organism?

A. Bacillus anthracis
B. Streptococcus pneumoniae
C. Staphylococcus aureus
D. Pseudomonas aeruginosa
E. Haemophilus influenzae




Category: Q&A Medicine--->Infectious Diseases
Page: 3 of 11